Riesz–Thorin theorem

From HandWiki
Short description: Theorem on operator interpolation

In mathematics, the Riesz–Thorin theorem, often referred to as the Riesz–Thorin interpolation theorem or the Riesz–Thorin convexity theorem, is a result about interpolation of operators. It is named after Marcel Riesz and his student G. Olof Thorin.

This theorem bounds the norms of linear maps acting between Lp spaces. Its usefulness stems from the fact that some of these spaces have rather simpler structure than others. Usually that refers to L2 which is a Hilbert space, or to L1 and L. Therefore one may prove theorems about the more complicated cases by proving them in two simple cases and then using the Riesz–Thorin theorem to pass from the simple cases to the complicated cases. The Marcinkiewicz theorem is similar but applies also to a class of non-linear maps.

Motivation

First we need the following definition:

Definition. Let p0, p1 be two numbers such that 0 < p0 < p1 ≤ ∞. Then for 0 < θ < 1 define pθ by: 1/pθ = 1 − θ/p0 + θ/p1.

By splitting up the function f in Lpθ as the product | f | = | f |1−θ | f |θ and applying Hölder's inequality to its pθ power, we obtain the following result, foundational in the study of Lp-spaces:

Proposition (log-convexity of Lp-norms) — Each f  ∈ Lp0Lp1 satisfies:

[math]\displaystyle{ \|f\|_{p_\theta} \leq \|f\|_{p_0}^{1-\theta}\|f\|_{p_1}^\theta. }[/math]

 

 

 

 

(1)

This result, whose name derives from the convexity of the map 1p ↦ log || f ||p on [0, ∞], implies that Lp0Lp1Lpθ.

On the other hand, if we take the layer-cake decomposition f  =  f1{|f|>1} +  f1{|f|≤1}, then we see that f1{|f|>1}Lp0 and f1{|f|≤1}Lp1, whence we obtain the following result:

Proposition — Each f in Lpθ can be written as a sum: f  = g + h, where gLp0 and hLp1.

In particular, the above result implies that Lpθ is included in Lp0 + Lp1, the sumset of Lp0 and Lp1 in the space of all measurable functions. Therefore, we have the following chain of inclusions:

Corollary — Lp0Lp1LpθLp0 + Lp1.

In practice, we often encounter operators defined on the sumset Lp0 + Lp1. For example, the Riemann–Lebesgue lemma shows that the Fourier transform maps L1(Rd) boundedly into L(Rd), and Plancherel's theorem shows that the Fourier transform maps L2(Rd) boundedly into itself, hence the Fourier transform [math]\displaystyle{ \mathcal{F} }[/math] extends to (L1 + L2) (Rd) by setting [math]\displaystyle{ \mathcal{F}(f_1+f_2) = \mathcal{F}_{L^1}(f_1) + \mathcal{F}_{L^2}(f_2) }[/math] for all f1  ∈ L1(Rd) and f2  ∈ L2(Rd). It is therefore natural to investigate the behavior of such operators on the intermediate subspaces Lpθ.

To this end, we go back to our example and note that the Fourier transform on the sumset L1 + L2 was obtained by taking the sum of two instantiations of the same operator, namely [math]\displaystyle{ \mathcal{F}_{L^1}:L^1(\mathbf{R}^d) \to L^\infty(\mathbf{R}^d), }[/math] [math]\displaystyle{ \mathcal{F}_{L^2}:L^2(\mathbf{R}^d) \to L^2(\mathbf{R}^d). }[/math]

These really are the same operator, in the sense that they agree on the subspace (L1L2) (Rd). Since the intersection contains simple functions, it is dense in both L1(Rd) and L2(Rd). Densely defined continuous operators admit unique extensions, and so we are justified in considering [math]\displaystyle{ \mathcal{F}_{L^1} }[/math] and [math]\displaystyle{ \mathcal{F}_{L^2} }[/math] to be the same.

Therefore, the problem of studying operators on the sumset Lp0 + Lp1 essentially reduces to the study of operators that map two natural domain spaces, Lp0 and Lp1, boundedly to two target spaces: Lq0 and Lq1, respectively. Since such operators map the sumset space Lp0 + Lp1 to Lq0 + Lq1, it is natural to expect that these operators map the intermediate space Lpθ to the corresponding intermediate space Lqθ.

Statement of the theorem

There are several ways to state the Riesz–Thorin interpolation theorem;[1] to be consistent with the notations in the previous section, we shall use the sumset formulation.

Riesz–Thorin interpolation theorem — Let 1, Σ1, μ1) and 2, Σ2, μ2) be σ-finite measure spaces. Suppose 1 ≤ p0 , q0 , p1 , q1 ≤ ∞, and let T : Lp0(μ1) + Lp1(μ1) → Lq0(μ2) + Lq1(μ2) be a linear operator that boundedly maps Lp0(μ1) into Lq0(μ2) and Lp1(μ1) into Lq1(μ2). For 0 < θ < 1, let pθ, qθ be defined as above. Then T boundedly maps Lpθ(μ1) into Lqθ(μ2) and satisfies the operator norm estimate

[math]\displaystyle{ \|T\|_{L^{p_\theta} \to L^{q_\theta}} \leq \|T\|^{1-\theta}_{L^{p_0} \to L^{q_0}} \|T\|^{\theta}_{L^{p_1} \to L^{q_1}}. }[/math]

 

 

 

 

(2)

In other words, if T is simultaneously of type (p0, q0) and of type (p1, q1), then T is of type (pθ, qθ) for all 0 < θ < 1. In this manner, the interpolation theorem lends itself to a pictorial description. Indeed, the Riesz diagram of T is the collection of all points (1/p, 1/q) in the unit square [0, 1] × [0, 1] such that T is of type (p, q). The interpolation theorem states that the Riesz diagram of T is a convex set: given two points in the Riesz diagram, the line segment that connects them will also be in the diagram.

The interpolation theorem was originally stated and proved by Marcel Riesz in 1927.[2] The 1927 paper establishes the theorem only for the lower triangle of the Riesz diagram, viz., with the restriction that p0q0 and p1q1. Olof Thorin extended the interpolation theorem to the entire square, removing the lower-triangle restriction. The proof of Thorin was originally published in 1938 and was subsequently expanded upon in his 1948 thesis.[3]

Proof

We will first prove the result for simple functions and eventually show how the argument can be extended by density to all measurable functions.

Simple Functions

By symmetry, let us assume [math]\displaystyle{ p_0 \lt p_1 }[/math] (the case [math]\displaystyle{ p_0 = p_1 }[/math] trivially follows from (1)). Let [math]\displaystyle{ f }[/math] be a simple function, that is [math]\displaystyle{ f = \sum_{j=1}^m a_j \mathbf{1}_{A_j} }[/math] for some finite [math]\displaystyle{ m\in\mathbb{N} }[/math], [math]\displaystyle{ a_j = \left\vert a_j\right\vert\mathrm{e}^{\mathrm{i}\alpha_j} \in \mathbb{C} }[/math] and [math]\displaystyle{ A_j\in\Sigma_1 }[/math], [math]\displaystyle{ j=1,2,\dots,m }[/math]. Similarly, let [math]\displaystyle{ g }[/math] denote a simple function [math]\displaystyle{ \Omega_2 \to \mathbb{C} }[/math], namely [math]\displaystyle{ g = \sum_{k=1}^n b_k \mathbf{1}_{B_k} }[/math] for some finite [math]\displaystyle{ n\in\mathbb{N} }[/math], [math]\displaystyle{ b_k = \left\vert b_k\right\vert\mathrm{e}^{\mathrm{i}\beta_k} \in \mathbb{C} }[/math] and [math]\displaystyle{ B_k\in\Sigma_2 }[/math], [math]\displaystyle{ k=1,2,\dots,n }[/math].

Note that, since we are assuming [math]\displaystyle{ \Omega_1 }[/math] and [math]\displaystyle{ \Omega_2 }[/math] to be [math]\displaystyle{ \sigma }[/math]-finite metric spaces, [math]\displaystyle{ f\in L^{r}(\mu_1) }[/math] and [math]\displaystyle{ g\in L^r(\mu_2) }[/math] for all [math]\displaystyle{ r \in [1, \infty] }[/math]. Then, by proper normalization, we can assume [math]\displaystyle{ \lVert f\rVert_{p_\theta}= 1 }[/math] and [math]\displaystyle{ \lVert g\rVert_{q_\theta'}=1 }[/math], with [math]\displaystyle{ q_\theta' = q_\theta(q_\theta-1)^{-1} }[/math] and with [math]\displaystyle{ p_\theta }[/math], [math]\displaystyle{ q_\theta }[/math] as defined by the theorem statement.

Next, we define the two complex functions [math]\displaystyle{ \begin{aligned} u: \mathbb{C}&\to \mathbb{C}& v: \mathbb{C}&\to \mathbb{C}\\ z &\mapsto u(z)=\frac{1-z}{p_0} + \frac{z}{p_1} & z &\mapsto v(z)=\frac{1-z}{q_0} + \frac{z}{q_1}.\end{aligned} }[/math] Note that, for [math]\displaystyle{ z=\theta }[/math], [math]\displaystyle{ u(\theta) = p_\theta^{-1} }[/math] and [math]\displaystyle{ v(\theta) = q_\theta^{-1} }[/math]. We then extend [math]\displaystyle{ f }[/math] and [math]\displaystyle{ g }[/math] to depend on a complex parameter [math]\displaystyle{ z }[/math] as follows: [math]\displaystyle{ \begin{aligned} f_z &= \sum_{j=1}^m \left\vert a_j\right\vert^{\frac{u(z)}{u(\theta)}} \mathrm{e}^{\mathrm{i}\alpha_j} \mathbf{1}_{A_j} \\ g_z &= \sum_{k=1}^n \left\vert b_k\right\vert^{\frac{1-v(z)}{1-v(\theta)}} \mathrm{e}^{\mathrm{i} \beta_k} \mathbf{1}_{B_k}\end{aligned} }[/math] so that [math]\displaystyle{ f_\theta = f }[/math] and [math]\displaystyle{ g_\theta = g }[/math]. Here, we are implicitly excluding the case [math]\displaystyle{ q_0 = q_1 = 1 }[/math], which yields [math]\displaystyle{ v\equiv 1 }[/math]: In that case, one can simply take [math]\displaystyle{ g_z=g }[/math], independently of [math]\displaystyle{ z }[/math], and the following argument will only require minor adaptations.

Let us now introduce the function [math]\displaystyle{ \Phi(z) = \int_{\Omega_2} (T f_z) g_z \,\mathrm{d}\mu_2 = \sum_{j=1}^m \sum_{k=1}^n \left\vert a_j\right\vert^{\frac{u(z)}{u(\theta)}} \left\vert b_k\right\vert^{\frac{1-v(z)}{1-v(\theta)}} \gamma_{j,k} }[/math] where [math]\displaystyle{ \gamma_{j,k} = \mathrm{e}^{\mathrm{i}(\alpha_j + \beta_k)} \int_{\Omega_2} (T \mathbf{1}_{A_j}) \mathbf{1}_{B_k} \,\mathrm{d}\mu_2 }[/math] are constants independent of [math]\displaystyle{ z }[/math]. We readily see that [math]\displaystyle{ \Phi(z) }[/math] is an entire function, bounded on the strip [math]\displaystyle{ 0 \le \operatorname{\mathbb{R}e}z \le 1 }[/math]. Then, in order to prove (2), we only need to show that

[math]\displaystyle{ \begin{aligned} \left\vert\Phi(\mathrm{i}y)\right\vert &\le \|T\|_{L^{p_0} \to L^{q_0}} &&\text{and} & \left\vert\Phi(1 + \mathrm{i}y)\right\vert &\le \|T\|_{L^{p_1} \to L^{q_1}}\end{aligned} }[/math]

 

 

 

 

(3)

for all [math]\displaystyle{ f_z }[/math] and [math]\displaystyle{ g_z }[/math] as constructed above. Indeed, if (3) holds true, by Hadamard three-lines theorem, [math]\displaystyle{ \left\vert\Phi(\theta + \mathrm{i}0)\right\vert = \biggl\vert\int_{\Omega_2} (Tf) g \,\mathrm{d}\mu_2\biggr\vert \le \|T\|_{L^{p_0} \to L^{q_0}}^{1-\theta} \|T\|_{L^{p_1} \to L^{q_1}}^\theta }[/math] for all [math]\displaystyle{ f }[/math] and [math]\displaystyle{ g }[/math]. This means, by fixing [math]\displaystyle{ f }[/math], that [math]\displaystyle{ \sup_g \biggl\vert\int_{\Omega_2} (Tf) g \,\mathrm{d}\mu_2\biggr\vert \le \|T\|_{L^{p_0} \to L^{q_0}}^{1-\theta} \|T\|_{L^{p_1} \to L^{q_1}}^\theta }[/math] where the supremum is taken with respect to all [math]\displaystyle{ g }[/math] simple functions with [math]\displaystyle{ \lVert g\rVert_{q_\theta'} = 1 }[/math]. The left-hand side can be rewritten by means of the following lemma.[4]

Lemma — Let [math]\displaystyle{ 1\le p, p' \le \infty }[/math] be conjugate exponents and let [math]\displaystyle{ f }[/math] be a function in [math]\displaystyle{ L^p(\mu_1) }[/math]. Then [math]\displaystyle{ \lVert f\rVert_p = \sup \biggl|\int_{\Omega_1} fg \,\mathrm{d}\mu_1\biggr| }[/math] where the supremum is taken over all simple functions [math]\displaystyle{ g }[/math] in [math]\displaystyle{ L^{p'}(\mu_1) }[/math] such that [math]\displaystyle{ \lVert g\rVert_{p'} \le 1 }[/math].

In our case, the lemma above implies [math]\displaystyle{ \lVert Tf\rVert_{q_\theta} \le \|T\|_{L^{p_0} \to L^{q_0}}^{1-\theta} \|T\|_{L^{p_1} \to L^{q_1}}^\theta }[/math] for all simple function [math]\displaystyle{ f }[/math] with [math]\displaystyle{ \lVert f\rVert_{p_\theta} = 1 }[/math]. Equivalently, for a generic simple function, [math]\displaystyle{ \lVert Tf\rVert_{q_\theta} \le \|T\|_{L^{p_0} \to L^{q_0}}^{1-\theta} \|T\|_{L^{p_1} \to L^{q_1}}^\theta \lVert f\rVert_{p_\theta}. }[/math]

Proof of (3)

Let us now prove that our claim (3) is indeed certain. The sequence [math]\displaystyle{ (A_j)_{j=1}^m }[/math] consists of disjoint subsets in [math]\displaystyle{ \Sigma_1 }[/math] and, thus, each [math]\displaystyle{ \xi\in \Omega_1 }[/math] belongs to (at most) one of them, say [math]\displaystyle{ A_{\hat{\jmath}} }[/math]. Then, for [math]\displaystyle{ z=\mathrm{i}y }[/math], [math]\displaystyle{ \begin{aligned} \left\vert f_{\mathrm{i}y}(\xi)\right\vert &= \left\vert a_{\hat{\jmath}}\right\vert^\frac{u(\mathrm{i}y)}{u(\theta)} \\ &= \exp\biggl(\log\left\vert a_{\hat{\jmath}}\right\vert\frac{p_\theta}{p_0}\biggr) \exp\biggl(-\mathrm{i}y \log\left\vert a_{\hat{\jmath}}\right\vert p_\theta\biggl(\frac{1}{p_0} - \frac{1}{p_1} \biggr) \biggr) \\ &= \left\vert a_{\hat{\jmath}}\right\vert^{\frac{p_\theta}{p_0}} \\ & = \left\vert f(\xi)\right\vert^{\frac{p_\theta}{p_0}}\end{aligned} }[/math] which implies that [math]\displaystyle{ \lVert f_{\mathrm{i}y}\rVert_{p_0} \le \lVert f\rVert_{p_\theta}^{\frac{p_\theta}{p_0}} }[/math]. With a parallel argument, each [math]\displaystyle{ \zeta \in \Omega_2 }[/math] belongs to (at most) one of the sets supporting [math]\displaystyle{ g }[/math], say [math]\displaystyle{ B_{\hat{k}} }[/math], and [math]\displaystyle{ \left\vert g_{\mathrm{i}y}(\zeta)\right\vert = \left\vert b_{\hat{k}}\right\vert^{\frac{1-1/q_0}{1-1/q_\theta}} = \left\vert g(\zeta)\right\vert^{\frac{1-1/q_0}{1-1/q_\theta}} = \left\vert g(\zeta)\right\vert^{\frac{q_\theta'}{q_0'}} \implies \lVert g_{\mathrm{i}y}\rVert_{q_0'} \le \lVert g\rVert_{q_\theta'}^{\frac{q_\theta'}{q_0'}}. }[/math]

We can now bound [math]\displaystyle{ \Phi(\mathrm{i}y) }[/math]: By applying Hölder’s inequality with conjugate exponents [math]\displaystyle{ q_0 }[/math] and [math]\displaystyle{ q_0' }[/math], we have [math]\displaystyle{ \begin{aligned} \left\vert\Phi(\mathrm{i}y)\right\vert &\le \lVert T f_{\mathrm{i}y}\rVert_{q_0} \lVert g_{\mathrm{i}y}\rVert_{q_0'} \\ &\le \|T\|_{L^{p_0} \to L^{q_0}} \lVert f_{\mathrm{i}y}\rVert_{p_0} \lVert g_{\mathrm{i}y}\rVert_{q_0'} \\ &= \|T\|_{L^{p_0} \to L^{q_0}} \lVert f\rVert_{p_\theta}^{\frac{p_\theta}{p_0}} \lVert g\rVert_{q_\theta'}^{\frac{q_\theta'}{q_0'}} \\ &= \|T\|_{L^{p_0} \to L^{q_0}}.\end{aligned} }[/math]

We can repeat the same process for [math]\displaystyle{ z=1+\mathrm{i}y }[/math] to obtain [math]\displaystyle{ \left\vert f_{1+\mathrm{i} y}(\xi)\right\vert = \left\vert f(\xi)\right\vert^{p_\theta/p_1} }[/math], [math]\displaystyle{ \left\vert g_{1+\mathrm{i}y}(\zeta)\right\vert = \left\vert g(\zeta)\right\vert^{q_\theta'/q_1'} }[/math] and, finally, [math]\displaystyle{ \left\vert\Phi(1+\mathrm{i}y)\right\vert \le \|T\|_{L^{p_1} \to L^{q_1}} \lVert f_{1+\mathrm{i}y}\rVert_{p_1} \lVert g_{1+\mathrm{i}y}\rVert_{q_1'} = \|T\|_{L^{p_1} \to L^{q_1}}. }[/math]

Extension to All Measurable Functions in Lpθ

So far, we have proven that

[math]\displaystyle{ \lVert Tf\rVert_{q_\theta} \le \|T\|_{L^{p_\theta} \to L^{q_\theta}} \lVert f\rVert_{p_\theta} }[/math]

 

 

 

 

(4)

when [math]\displaystyle{ f }[/math] is a simple function. As already mentioned, the inequality holds true for all [math]\displaystyle{ f\in L^{p_\theta}(\Omega_1) }[/math] by the density of simple functions in [math]\displaystyle{ L^{p_\theta}(\Omega_1) }[/math].

Formally, let [math]\displaystyle{ f\in L^{p_\theta}(\Omega_1) }[/math] and let [math]\displaystyle{ (f_n)_n }[/math] be a sequence of simple functions such that [math]\displaystyle{ \left\vert f_n\right\vert \le \left\vert f\right\vert }[/math], for all [math]\displaystyle{ n }[/math], and [math]\displaystyle{ f_n \to f }[/math] pointwise. Let [math]\displaystyle{ E=\{x\in \Omega_1: \left\vert f(x)\right\vert \gt 1\} }[/math] and define [math]\displaystyle{ g = f \mathbf{1}_E }[/math], [math]\displaystyle{ g_n = f_n \mathbf{1}_E }[/math], [math]\displaystyle{ h = f - g = f \mathbf{1}_{E^\mathrm{c}} }[/math] and [math]\displaystyle{ h_n = f_n - g_n }[/math]. Note that, since we are assuming [math]\displaystyle{ p_0 \le p_\theta \le p_1 }[/math], [math]\displaystyle{ \begin{aligned} \lVert f\rVert_{p_\theta}^{p_\theta} &= \int_{\Omega_1} \left\vert f\right\vert^{p_\theta} \,\mathrm{d}\mu_1 \ge \int_{\Omega_1} \left\vert f\right\vert^{p_\theta} \mathbf{1}_{E} \,\mathrm{d}\mu_1 \ge \int_{\Omega_1} \left\vert f \mathbf{1}_{E}\right\vert^{p_0} \,\mathrm{d}\mu_1 = \int_{\Omega_1} \left\vert g\right\vert^{p_0} \,\mathrm{d}\mu_1 = \lVert g\rVert_{p_0}^{p_0} \\ \lVert f\rVert_{p_\theta}^{p_\theta} &= \int_{\Omega_1} \left\vert f\right\vert^{p_\theta} \,\mathrm{d}\mu_1 \ge \int_{\Omega_1} \left\vert f\right\vert^{p_\theta} \mathbf{1}_{E^\mathrm{c}} \,\mathrm{d}\mu_1 \ge \int_{\Omega_1} \left\vert f \mathbf{1}_{E^\mathrm{c}}\right\vert^{p_1} \,\mathrm{d}\mu_1 = \int_{\Omega_1} \left\vert h\right\vert^{p_1} \,\mathrm{d}\mu_1 = \lVert h\rVert_{p_1}^{p_1}\end{aligned} }[/math] and, equivalently, [math]\displaystyle{ g\in L^{p_0}(\Omega_1) }[/math] and [math]\displaystyle{ h\in L^{p_1}(\Omega_1) }[/math].

Let us see what happens in the limit for [math]\displaystyle{ n\to\infty }[/math]. Since [math]\displaystyle{ \left\vert f_n\right\vert \le \left\vert f\right\vert }[/math], [math]\displaystyle{ \left\vert g_n\right\vert \le \left\vert g\right\vert }[/math] and [math]\displaystyle{ \left\vert h_n\right\vert \le \left\vert h\right\vert }[/math], by the dominated convergence theorem one readily has [math]\displaystyle{ \begin{aligned} \lVert f_n\rVert_{p_\theta} &\to \lVert f\rVert_{p_\theta} & \lVert g_n\rVert_{p_0} &\to \lVert g\rVert_{p_0} & \lVert h_n\rVert_{p_1} &\to \lVert h\rVert_{p_1}.\end{aligned} }[/math] Similarly, [math]\displaystyle{ \left\vert f - f_n\right\vert \le 2\left\vert f\right\vert }[/math], [math]\displaystyle{ \left\vert g-g_n\right\vert \le 2\left\vert g\right\vert }[/math] and [math]\displaystyle{ \left\vert h - h_n\right\vert \le 2\left\vert h\right\vert }[/math] imply [math]\displaystyle{ \begin{aligned} \lVert f - f_n\rVert_{p_\theta} &\to 0 & \lVert g - g_n\rVert_{p_0} &\to 0 & \lVert h - h_n\rVert_{p_1} &\to 0\end{aligned} }[/math] and, by the linearity of [math]\displaystyle{ T }[/math] as an operator of types [math]\displaystyle{ (p_0, q_0) }[/math] and [math]\displaystyle{ (p_1, q_1) }[/math] (we have not proven yet that it is of type [math]\displaystyle{ (p_\theta, q_\theta) }[/math] for a generic [math]\displaystyle{ f }[/math]) [math]\displaystyle{ \begin{aligned} \lVert Tg - Tg_n\rVert_{p_0} & \le \|T\|_{L^{p_0} \to L^{q_0}} \lVert g - g_n\rVert_{p_0} \to 0 & \lVert Th - Th_n\rVert_{p_1} & \le \|T\|_{L^{p_1} \to L^{q_1}} \lVert h - h_n\rVert_{p_1} \to 0.\end{aligned} }[/math]

It is now easy to prove that [math]\displaystyle{ Tg_n \to Tg }[/math] and [math]\displaystyle{ Th_n \to Th }[/math] in measure: For any [math]\displaystyle{ \epsilon \gt 0 }[/math], Chebyshev’s inequality yields [math]\displaystyle{ \mu_2(y\in \Omega_2: \left\vert Tg - Tg_n\right\vert \gt \epsilon) \le \frac{\lVert Tg - Tg_n\rVert_{q_0}^{q_0}} {\epsilon^{q_0}} }[/math] and similarly for [math]\displaystyle{ Th - Th_n }[/math]. Then, [math]\displaystyle{ Tg_n \to Tg }[/math] and [math]\displaystyle{ Th_n \to Th }[/math] a.e. for some subsequence and, in turn, [math]\displaystyle{ Tf_n \to Tf }[/math] a.e. Then, by Fatou’s lemma and recalling that (4) holds true for simple functions, [math]\displaystyle{ \lVert Tf\rVert_{q_\theta} \le \liminf_{n\to\infty} \lVert T f_n\rVert_{q_\theta} \le \|T\|_{L^{p_\theta} \to L^{q_\theta}} \liminf_{n\to\infty} \lVert f_n\rVert_{p_\theta} = \|T\|_{L^{p_\theta} \to L^{q_\theta}} \lVert f\rVert_{p_\theta}. }[/math]

Interpolation of analytic families of operators

The proof outline presented in the above section readily generalizes to the case in which the operator T is allowed to vary analytically. In fact, an analogous proof can be carried out to establish a bound on the entire function [math]\displaystyle{ \varphi(z) = \int (T_z f_z)g_z \, d\mu_2, }[/math] from which we obtain the following theorem of Elias Stein, published in his 1956 thesis:[5]

Stein interpolation theorem — Let 1, Σ1, μ1) and 2, Σ2, μ2) be σ-finite measure spaces. Suppose 1 ≤ p0 , p1 ≤ ∞, 1 ≤ q0 , q1 ≤ ∞, and define:

S = {zC : 0 < Re(z) < 1},
S = {zC : 0 ≤ Re(z) ≤ 1}.

We take a collection of linear operators {Tz : zS} on the space of simple functions in L1(μ1) into the space of all μ2-measurable functions on Ω2. We assume the following further properties on this collection of linear operators:

  • The mapping [math]\displaystyle{ z \mapsto \int (T_zf)g \, d\mu_2 }[/math] is continuous on S and holomorphic on S for all simple functions f and g.
  • For some constant k < π, the operators satisfy the uniform bound: [math]\displaystyle{ \sup_{z \in S} e^{-k|\text{Im}(z)|} \log \left| \int (T_zf)g \, d\mu_2 \right| \lt \infty }[/math]
  • Tz maps Lp0(μ1) boundedly to Lq0(μ2) whenever Re(z) = 0.
  • Tz maps Lp1(μ1) boundedly to Lq1(μ2) whenever Re(z) = 1.
  • The operator norms satisfy the uniform bound [math]\displaystyle{ \sup_{\text{Re}(z) = 0, 1} e^{-k|\text{Im}(z)|} \log \left \|T_z \right \| \lt \infty }[/math] for some constant k < π.

Then, for each 0 < θ < 1, the operator Tθ maps Lpθ(μ1) boundedly into Lqθ(μ2).

The theory of real Hardy spaces and the space of bounded mean oscillations permits us to wield the Stein interpolation theorem argument in dealing with operators on the Hardy space H1(Rd) and the space BMO of bounded mean oscillations; this is a result of Charles Fefferman and Elias Stein.[6]

Applications

Hausdorff–Young inequality

Main page: Hausdorff–Young inequality

It has been shown in the first section that the Fourier transform [math]\displaystyle{ \mathcal{F} }[/math] maps L1(Rd) boundedly into L(Rd) and L2(Rd) into itself. A similar argument shows that the Fourier series operator, which transforms periodic functions f  : TC into functions [math]\displaystyle{ \hat{f}:\mathbf{Z} \to \mathbf{C} }[/math] whose values are the Fourier coefficients [math]\displaystyle{ \hat{f}(n) = \frac{1}{2\pi} \int_{-\pi}^{\pi} f(x) e^{-inx} \, dx , }[/math] maps L1(T) boundedly into (Z) and L2(T) into 2(Z). The Riesz–Thorin interpolation theorem now implies the following: [math]\displaystyle{ \begin{align} \left \|\mathcal{F}f \right \|_{L^{q}(\mathbf{R}^d)} &\leq \|f\|_{L^p(\mathbf{R}^d)} \\ \left \|\hat{f} \right \|_{\ell^{q}(\mathbf{Z})} &\leq \|f\|_{L^p(\mathbf{T})} \end{align} }[/math] where 1 ≤ p ≤ 2 and 1/p + 1/q = 1. This is the Hausdorff–Young inequality.

The Hausdorff–Young inequality can also be established for the Fourier transform on locally compact Abelian groups. The norm estimate of 1 is not optimal. See the main article for references.

Convolution operators

Main page: Young's convolution inequality

Let f be a fixed integrable function and let T be the operator of convolution with f, i.e., for each function g we have Tg =  f  ∗ g.

It is well known that T is bounded from L1 to L1 and it is trivial that it is bounded from L to L (both bounds are by || f ||1). Therefore the Riesz–Thorin theorem gives [math]\displaystyle{ \| f * g \|_p \leq \|f\|_1 \|g\|_p. }[/math]

We take this inequality and switch the role of the operator and the operand, or in other words, we think of S as the operator of convolution with g, and get that S is bounded from L1 to Lp. Further, since g is in Lp we get, in view of Hölder's inequality, that S is bounded from Lq to L, where again 1/p + 1/q = 1. So interpolating we get [math]\displaystyle{ \|f*g\|_s\leq \|f\|_r\|g\|_p }[/math] where the connection between p, r and s is [math]\displaystyle{ \frac{1}{r}+\frac{1}{p}=1+\frac{1}{s}. }[/math]

The Hilbert transform

Main page: Hilbert transform

The Hilbert transform of f  : RC is given by [math]\displaystyle{ \mathcal{H}f(x) = \frac{1}{\pi} \, \mathrm{p.v.} \int_{-\infty}^\infty \frac{f(x-t)}{t} \, dt = \left(\frac{1}{\pi} \, \mathrm{p.v.} \frac{1}{t} \ast f\right)(x), }[/math] where p.v. indicates the Cauchy principal value of the integral. The Hilbert transform is a Fourier multiplier operator with a particularly simple multiplier: [math]\displaystyle{ \widehat{\mathcal{H}f}(\xi) = -i \, \sgn(\xi) \hat{f}(\xi). }[/math]

It follows from the Plancherel theorem that the Hilbert transform maps L2(R) boundedly into itself.

Nevertheless, the Hilbert transform is not bounded on L1(R) or L(R), and so we cannot use the Riesz–Thorin interpolation theorem directly. To see why we do not have these endpoint bounds, it suffices to compute the Hilbert transform of the simple functions 1(−1,1)(x) and 1(0,1)(x) − 1(0,1)(−x). We can show, however, that [math]\displaystyle{ (\mathcal{H}f)^2 = f^2 + 2\mathcal{H}(f\mathcal{H}f) }[/math] for all Schwartz functions f  : RC, and this identity can be used in conjunction with the Cauchy–Schwarz inequality to show that the Hilbert transform maps L2n(Rd) boundedly into itself for all n ≥ 2. Interpolation now establishes the bound [math]\displaystyle{ \|\mathcal{H}f\|_p \leq A_p \|f\|_p }[/math] for all 2 ≤ p < ∞, and the self-adjointness of the Hilbert transform can be used to carry over these bounds to the 1 < p ≤ 2 case.

Comparison with the real interpolation method

While the Riesz–Thorin interpolation theorem and its variants are powerful tools that yield a clean estimate on the interpolated operator norms, they suffer from numerous defects: some minor, some more severe. Note first that the complex-analytic nature of the proof of the Riesz–Thorin interpolation theorem forces the scalar field to be C. For extended-real-valued functions, this restriction can be bypassed by redefining the function to be finite everywhere—possible, as every integrable function must be finite almost everywhere. A more serious disadvantage is that, in practice, many operators, such as the Hardy–Littlewood maximal operator and the Calderón–Zygmund operators, do not have good endpoint estimates.[7] In the case of the Hilbert transform in the previous section, we were able to bypass this problem by explicitly computing the norm estimates at several midway points. This is cumbersome and is often not possible in more general scenarios. Since many such operators satisfy the weak-type estimates [math]\displaystyle{ \mu \left( \{x : Tf(x) \gt \alpha \} \right) \leq \left( \frac{C_{p,q} \|f\|_p}{\alpha} \right)^q, }[/math] real interpolation theorems such as the Marcinkiewicz interpolation theorem are better-suited for them. Furthermore, a good number of important operators, such as the Hardy-Littlewood maximal operator, are only sublinear. This is not a hindrance to applying real interpolation methods, but complex interpolation methods are ill-equipped to handle non-linear operators. On the other hand, real interpolation methods, compared to complex interpolation methods, tend to produce worse estimates on the intermediate operator norms and do not behave as well off the diagonal in the Riesz diagram. The off-diagonal versions of the Marcinkiewicz interpolation theorem require the formalism of Lorentz spaces and do not necessarily produce norm estimates on the Lp-spaces.

Mityagin's theorem

B. Mityagin extended the Riesz–Thorin theorem; this extension is formulated here in the special case of spaces of sequences with unconditional bases (cf. below).

Assume: [math]\displaystyle{ \|A\|_{\ell_1 \to \ell_1}, \|A\|_{\ell_\infty \to \ell_\infty} \leq M. }[/math]

Then [math]\displaystyle{ \|A\|_{X \to X} \leq M }[/math]

for any unconditional Banach space of sequences X, that is, for any [math]\displaystyle{ (x_i) \in X }[/math] and any [math]\displaystyle{ (\varepsilon_i) \in \{-1, 1 \}^\infty }[/math], [math]\displaystyle{ \| (\varepsilon_i x_i) \|_X = \| (x_i) \|_X }[/math].

The proof is based on the Krein–Milman theorem.

See also

Notes

  1. Stein and Weiss (1971) and Grafakos (2010) use operators on simple functions, and Muscalu and Schlag (2013) uses operators on generic dense subsets of the intersection Lp0Lp1. In contrast, Duoanddikoetxea (2001), Tao (2010), and Stein and Shakarchi (2011) use the sumset formulation, which we adopt in this section.
  2. Riesz (1927). The proof makes use of convexity results in the theory of bilinear forms. For this reason, many classical references such as Stein and Weiss (1971) refer to the Riesz–Thorin interpolation theorem as the Riesz convexity theorem.
  3. Thorin (1948)
  4. Bernard, Calista. "Interpolation theorems and applications". http://math.uchicago.edu/~may/REU2013/REUPapers/Bernard.pdf. 
  5. Stein (1956). As Charles Fefferman points out in his essay in Fefferman, Fefferman, Wainger (1995), the proof of Stein interpolation theorem is essentially that of the Riesz–Thorin theorem with the letter z added to the operator. To compensate for this, a stronger version of the Hadamard three-lines theorem, due to Isidore Isaac Hirschman, Jr., is used to establish the desired bounds. See Stein and Weiss (1971) for a detailed proof, and a blog post of Tao for a high-level exposition of the theorem.
  6. Fefferman and Stein (1972)
  7. Elias Stein is quoted for saying that interesting operators in harmonic analysis are rarely bounded on L1 and L.

References

  • Dunford, N.; Schwartz, J.T. (1958), Linear operators, Parts I and II, Wiley-Interscience .
  • Fefferman, Charles; Stein, Elias M. (1972), "[math]\displaystyle{ H^p }[/math] Spaces of Several variables", Acta Mathematica 129: 137–193, doi:10.1007/bf02392215 
  • Glazman, I.M.; Lyubich, Yu.I. (1974), Finite-dimensional linear analysis: a systematic presentation in problem form, Cambridge, Mass.: The M.I.T. Press . Translated from the Russian and edited by G. P. Barker and G. Kuerti.
  • Hörmander, L. (1983), The analysis of linear partial differential operators I, Grundl. Math. Wissenschaft., 256, Springer, doi:10.1007/978-3-642-96750-4, ISBN 3-540-12104-8 .
  • Mitjagin [Mityagin], B.S. (1965), "An interpolation theorem for modular spaces (Russian)", Mat. Sb., New Series 66 (108): 473–482 .
  • Thorin, G. O. (1948), "Convexity theorems generalizing those of M. Riesz and Hadamard with some applications", Comm. Sem. Math. Univ. Lund [Medd. Lunds Univ. Mat. Sem.] 9: 1–58 
  • Riesz, Marcel (1927), "Sur les maxima des formes bilinéaires et sur les fonctionnelles linéaires", Acta Mathematica 49 (3–4): 465–497, doi:10.1007/bf02564121 
  • Stein, Elias M. (1956), "Interpolation of Linear Operators", Trans. Amer. Math. Soc. 83 (2): 482–492, doi:10.1090/s0002-9947-1956-0082586-0 
  • Stein, Elias M.; Shakarchi, Rami (2011), Functional Analysis: Introduction to Further Topics in Analysis, Princeton University Press 
  • Stein, Elias M.; Weiss, Guido (1971), Introduction to Fourier Analysis on Euclidean Spaces, Princeton University Press 

External links